LSAT and Law School Admissions Forum

Get expert LSAT preparation and law school admissions advice from PowerScore Test Preparation.

 Administrator
PowerScore Staff
  • PowerScore Staff
  • Posts: 8917
  • Joined: Feb 02, 2011
|
#98368
Complete Question Explanation

Weaken. The correct answer choice is (C).

Answer choice (A):

Answer choice (B):

Answer choice (C): This is the correct answer choice.

Answer choice (D):

Answer choice (E):

This explanation is still in progress. Please post any questions below!
 sarah_tucker@alumni.brown.edu
  • Posts: 17
  • Joined: Sep 20, 2021
|
#98991
I chose A because it seemed that having inconvenient hours would not meet resident needs. Is A incorrect because the branch hours do not necessarily reflect the hours of the new, larger library?
 Luke Haqq
PowerScore Staff
  • PowerScore Staff
  • Posts: 742
  • Joined: Apr 26, 2012
|
#99013
Hi Sarah!

To see why (A) is incorrect and (C) correct, take with the conclusion of the stimulus: "Thus, Winchester would better meet its residents' needs by closing its small branch libraries and opening one larger, well-supplied library."

Answer choice (A) states that "Many Winchester residents never use the branch libraries because of the irregular hours each branch keeps." This answer choice seems problematic because it is a conclusion about the branch libraries, when the conclusion of the stimulus is about a large library better meeting residents' needs. This answer choice doesn't mention the one, larger library, so it's unclear how it could weaken a conclusion about that type of library.

Instead, answer choice (A) might even strengthen the conclusion. The conclusion is in favor of one larger library rather than several small branches. If many people never used the branch libraries because of irregular hours in the first place, this seems to strengthen the argument when the question stem instead asks for an answer choice that weakens it.

Answer choice (C) states, "The only possible site for a new library is not readily accessible to most Winchester residents." When I read the stimulus, a prephrase that came to mind was that branch libraries dispersed throughout Winchester could more easily reach everyone, whereas one central library might be too far away for many or most Winchester residents to want to use it. Answer choice (C) effectively says the same thing, that one large library might not better serve Winchester residents because it would be on a site that isn't readily accessible to most residents.

Get the most out of your LSAT Prep Plus subscription.

Analyze and track your performance with our Testing and Analytics Package.